What's the answer for this? I can't find the answer. ​

What's The Answer For This? I Can't Find The Answer.

Answers

Answer 1

I believe the answer is 3/4.

Answer 2

Answer:

1/4+2/4=3/4 I believe so


Related Questions

There are not asymptotes for the parent function
f (x) =1/x
True
False

Answers

Answer:

True

Step-by-step explanation:

the parent function f(x) = 1/x has a vertical asymptote at x = 0 and a horizontal asymptote at y = 0

HAS TO BE DONE BY TOMORROW - A positive integer is called snakelike if its digits alternately increase and decrease or vice versa. For example, 130 and 91727 are snakelike. However 123 is not snakelike (2 is greater than 1 but not 3) and 91727 is not snakelike (7 is greater than 1 but not 7) A) How many 4-digit snakelike numbers contain each of the digit 1,2,3,4 exactly once B) How many 3-Digit snakelike numbers have 6 as their middle digit C) a 7-digit snakelike number starts with digit 9. Explain why it last digit cannot be 0

Answers

Answer:

A)

The simplest way to solve A is by checking 1st digit one-by-one.

selecting 1 as 1st digit, we have 2 ways

1324 and 1423

selecting 2 as 1st digit, we have 3 ways

2143, 2314, and 2413

selecting 3 as 1st digit, we have 3 ways

3142, 3241, and 3412

selecting 4 as 1st digit, we have 2 ways

4132 and 4231

=> 2 +3 + 3 + 2 = 10 ways in total.

B)

Use the same strategy as A)

selecting 1st digit that is greater than 6, there are 3 digits: {9, 8, 7}

selecting 9 as 1st digit => 967, 968

selecting 8 as 1st digit => 867, 869

selecting 7 as 1st digit => 768, 769

=> 3 x 2 = 6 ways

selecting 1st digit that is smaller than 6, there are 5 digits: {5, 4, 3, 2, 1}

selecting 5 as 1st digit => 565, 564, 563, 562, 561, 560

similarly, selecting 4, 3, 2, 1, there are 6 ways to select that last digit

=> 5 x 6 = 30 ways

=> 30 + 6 = 36 ways in total

C)

a 7-digit snakelike number starts with digit 9 (the largest digit)

=> 2nd digit is smaller than 9

=> 3rd digit is larger than 2nd digit

=> 4th digit is smaller than 3rd digit

=> 5th digit is larger than 4th digit

=> 6th digit is smaller than 5th digit

=> 7th digit is larger than 6th digit (this is impossible because 0 is smallest digit)

Four-thirds (x) minus one-third = 9. Four-thirds (x) minus one-third + one-third = 9 + one-third. Four-thirds (x) = StartFraction 28 over 3 EndFraction

Answers

Answer:

[tex]x = 7[/tex]

Step-by-step explanation:

Given

[tex]\frac{4}{3}x - \frac{1}{3} = 9[/tex]

Required

Solve for x

[tex]\frac{4}{3}x - \frac{1}{3} = 9[/tex]

Add [tex]\frac{1}{3}[/tex] to both sides

[tex]\frac{4}{3}x - \frac{1}{3} + \frac{1}{3} = 9 + \frac{1}{3}[/tex]

[tex]\frac{4}{3}x = 9 + \frac{1}{3}[/tex]

Take LCM

[tex]\frac{4}{3}x = \frac{27 + 1}{3}[/tex]

[tex]\frac{4}{3}x = \frac{28}{3}[/tex]

Multiply both sides by [tex]\frac{3}{4}[/tex]

[tex]\frac{3}{4} * \frac{4}{3}x = \frac{3}{4} *\frac{28}{3}[/tex]

Evaluate the left hand side

[tex]x = \frac{3}{4} *\frac{28}{3}[/tex]

[tex]x = \frac{28}{4}[/tex]

Divide 28 by 4

[tex]x = 7[/tex]

Answer:

x=7 we are both correct

Step-by-step explanation:

Solve this equation. 2(3x+5)=4(2x-3)

Answers

Answer:

x = 11

Step-by-step explanation:

2(3x+5) = 4(2x - 3)

First expand the terms

That's

6x + 10 = 8x - 12

6x - 8x + 10 = - 12

- 2x + 10 = - 12

Subtract 10 from both sides

That's

- 2x + 10 - 10 = - 12 - 10

- 2x = - 22

Divide both sides by - 2

We have the final answer as

x = 11

Hope this helps you

6x+10=8x-12
6x-8x=-12-10
-2x=-22
x=11

Hope this can help.

63001 square root by division method​

Answers

Answer:

this is a correct answer

Help me do this please

Answers

Answer:

r = - 2

Step-by-step explanation:

Use the slope formula to find m then equate to - 1

m = [tex]\frac{y_{2}-y_{1} }{x_{2}-x_{1} }[/tex]

with (x₁, y₁ ) = (6, r) and (x₂, y₂ ) = (9, - 5)

m = [tex]\frac{-5-r}{9-6}[/tex] = [tex]\frac{-5-r}{3}[/tex]  = - 1 ( multiply both sides by 3 )

- 5 - r = - 3 ( add 5 to both sides )

- r = 2 ( multiply both sides by - 1 )

r = - 2

the sum of a number and -43 is 102.what is the number ​

Answers

Answer: 145

Step-by-step explanation:

Answer: 145

Step by step explanation:

what is the simplified value of the expression -2.6+5.8​

Answers

Answer:

[See Below]

Step-by-step explanation:

✦ Solve:

  ✧ -2.6 + 5.8 = 3.2

✦ Sometimes in math simplify could mean just solve and get the smallest version of that number. That's what it means in this case. So the simplified version would be 3.2.

~Hope this helps Mate. Feel free to message me if you need anything.

solve for n. n-5•2/3=3/4

Answers

Exact form is N=49/12 mixed number form is N=4 1/12

Step-by-step explanation:you have to simplify both sides and then isolate the variable.

Answer:

n=49/12 or 4 1/12

Step-by-step explanation:

n-5•2/3=3/4

n-10/3=3/4

n=3/4+10/3

n=49/12 or 4 1/12

please i need help like right now

Answers

I have no idea what to do. We have to do the same stuff too

Simplify (6-2)(33)(3-6)

Answers

Answer:

-396

Step-by-step explanation:

Plug it into a calculator

4 x 33 = 132

132 x -3 = -396

4 x 33 x -3= 132 x -3= -396

Using the remainder theorem, which quotient has a remainder of 20?
OA (3r4 – 573 + 5x + 2) = (x - 2)
O B. (3r3 + 702 + 5x + 2) = (x - 2)
Oc. (13 + 5x2 4r + 6) = (x + 5)
OD. (74 – 812 + 16) = (x + 4)

Answers

Answer:

The answer is A. (3x^4-5x^3+5x+2) =(x-2)

Step-by-step explanation:

we see that the polynomial that gives the remainder as 20 is

(3[tex]x^{4}[/tex] - 5x³ + 5x + 2) ÷ (x - 2)

Option A is our correct answer.

What is the remainder theorem?

The remainder theorem states that when a polynomial p(x) is divided by

(x - a), then the remainder = f(a).

We have,

(3[tex]x^{4}[/tex] - 5x³ + 5x + 2) ÷ (x - 2) ____(1)

We can directly find the remainder as:

x - 2 = 0

x = 2

Putting x = 2 in (1) we get,

= 3 x 16 - 5 x 8 + 5 x 2 + 2

= 48 - 40 + 10 + 2

= 20

The remainder is 20.

(3x³ + 7x² + 5x + 2) ÷ (x - 2) ) _______(2)

x - 2 = 0

x = 2

Putting x = 2 in (2) we get,

= 3 x 8 + 7 x 4 + 5 x 2 + 2

= 24 + 28 + 10 + 2

= 64

The remainder is 64.

(x³ + 5x² - 4x + 6) ÷ (x + 5) ______(3)

x + 5 = 0

x = -5

Putting x = -5 in (3) we get,

= - 125 + 5 x 25 - 4 x (-5) + 6

= -125 + 125 + 20 + 6

= 20 + 6

= 26

The remainder is 26.

([tex]x^{4}[/tex] - 8x² + 16) ÷ (x + 4) _______(4)

x + 4 = 0

x = -4

Putting x = -4 in (4) we get,

= 256 - 8 x 16 + 16

= 256 - 128 + 16

= 144

The remainder is 144.

Thus we see that the polynomial that gives the remainder as 20 is

(3[tex]x^{4}[/tex] - 5x³ + 5x + 2) ÷ (x - 2)

Option A is our correct answer.

Learn more about the remainder theorem here:

https://brainly.com/question/9160275

#SPJ5

PLEASE HELP!!!!There is an 80% chance of rain. The tennis team has a 75% chance of winning if it doesn't rain and only a 35% chance of winning if it does rain. Make a tree diagram with the following probabilities. ~Will mark brain

Answers

Answer:

00304761

Step-by-step explanation:

What is being constructed based in the markings in the following diagram?

Answers

Answer:

Perpendicular line through a point off a line

Step-by-step explanation:

Based on the markings in the diagram, we have;

Marking points E and F along AB extended forms two points equidistant from the point C on the line AB

Construction of the line HI by using equal arc lengths from E and F creates two points equidistant from E and F on either side of the line AB such that HI passes through C forming a perpendicular bisector from C to AB.

Therefore, a perpendicular line has been constructed from the point C off the line AB to the line AB

Hence the correct option is the construction of a perpendicular line through a point off a line.

Answer:

C. Perpendicular line through a point off a line (its off not on)  Took the test and got it right

Step-by-step explanation:

Show that each function is a quadratic function by writing it in the form
f(x) = ax² + bx+c and Identify a, b, and c.

1. h(x) = 2(x + 1)(3x – 4)

Answers

Answer:

h(x) = 2x² - 2x - 8

Step-by-step explanation:

Step 1: Write out equation

h(x) = 2(x + 1)(3x - 4)

Step 2: FOIL

h(x) = 2(x² - 4x + 3x - 4)

Step 3: Combine like terms (x)

h(x) = 2(x² - x - 4)

Step 4: Distribute

h(x) = 2x² - 2x - 8

Red pencils cost 3 for 10¢. Blue pencils cost 5¢ each. Green pencils cost 2 for 5¢. If you can purchase any color you wish, what is the greatest number of pencils you can you can buy for 50¢?

Answers

Answer: 20 green pencils

Step-by-step explanation:

given data:

cost of red pencils = 3 for 10¢.

cost of green pencils = 2 for 5¢.

cost of blue pencils = 5¢/each.

Greatest number of pencils one can buy for 50¢ By choosi any color is 20pencils which is green

red pencils 50¢ = 15 pencils

green pencils 50¢  = 20 pencils

blue pencils 50¢  = 10 pencils

Answer:

20

Step-by-step explanation:

Help me solve this Question thanks

Answers

Answer:

C

Step-by-step explanation:

Given

[tex]\frac{-m}{2}[/tex] - [tex]\frac{5}{4}[/tex] ≤ [tex]\frac{5m}{12}[/tex] - [tex]\frac{7}{6}[/tex]

Multiply through by 12 to clear the fractions

- 6m - 15 ≤ 5m - 14 ( subtract 5m from both sides )

- 11m - 15 ≤ - 14 ( add 15 to both sides )

- 11m ≤ 1

Divide both sides by - 11, reversing the symbol as a result of dividing by a negative quantity

m ≥ - [tex]\frac{1}{11}[/tex] → C

Which sequences are arithmetic? Select three options.

–8.6, –5.0, –1.4, 2.2, 5.8, …
2, –2.2, 2.42, –2.662, 2.9282, …
5, 1, –3, –7, –11, …
–3, 3, 9, 15, 21, …
–6.2, –3.1, –1.55, –0.775, –0.3875, …

Answers

Answer:

it should be 2nd option, 3rd, and 5th.

Step-by-step explanation:

From the given options

options I, III and IV  are Arithmetic sequences.

Given :

we are given with sequences. we need to check which one is arithmetic sequence.

A sequence is arithmetic when the consecutive terms have common difference.

Lets check one by one

[tex]-8.6, -5.0, -1.4, 2.2, 5.8,......\\-5-(-8.6)=3.6\\-1.4-(-5)=3.6\\2.2+1.4=3.6\\[/tex]

Its arithmetic because common difference is same 3.6

2, –2.2, 2.42, –2.662, 2.9282, …

-2.2-2=-4.2

2.42+2.2=6.62

Its not arithmetic

5, 1, –3, –7, –11, …

1-5=-4

-3-1=-4

-7+3=-4

-11+7=-3

Common difference is -4. So it Arithmetic

–3, 3, 9, 15, 21, …

3+3=6

9-3=6

15-9=6

Its Arithmetic

–6.2, –3.1, –1.55, –0.775, –0.3875, …

-3.1+6.2=3.1

-1.55+3.1=1.55

Its not Arithmetic

So, options I, III and IV  are Arithmetic sequences.

Learn more :  brainly.com/question/12870930

5(x+7)=10
Ghggggggggggggggg

Answers

Answer:

x=-5

Step-by-step explanation:

x=10/5 - 7

x=2-7

x=-5

4(2x-1)=3x+5-x do y’all know the answer ?

Answers

Answer:

Step-by-step explanation:

4(2x-1)=3x+5-x

8x-4=3x+5-x

8x-4=2x+5

6x-4=5

6x=9

x=3/2

Answer: x=1.3

Explanation: Just simplify the equation so there is a variable on one side and a number on the other.

4. a) In a sale, a car costing $ 9500 Średuced by
30%
what's the sale Price?​

Answers

Answer:

$6650

Step-by-step explanation:

→ Calculate the decimal multiplier

100 - 30 = 70

70 ÷ 100 = 0.7

→ Multiply the decimal multiplier by the cost

9500 × 0.7 = 6650

Answer:

$6,650

Step-by-step explanation:

This is called percentage decrease:

(100 - 30)% = 70%

9500 × 70/100

= $6,650


The question i’m asking is if you can help me get 2 equations that if you type on a graph you will get the point (2,5)

Answers

Answer: The equations are y = 4x-3  and y= 3x -1

Step-by-step explanation:

So lets say both equations will have positive slopes. Using that, we can input any slope  and solve for the y intercept using the coordinates.We will use the formula, y=mx + b  where m is the slope and b is the y-intercept

Let's say the slope is 3.

5 = 3(2) + b

5= 6 + b

-6  -6  

b= -1  

We can now come up with the equation, y= 3x -1  

We need to find another equation that will intersect with it at the point (2,5)

For lines to intersect, they need to have different slopes and different y-intercepts.

We need to come up with a new slope, the slope of the second equation could be 4. Now using the formula, use the coordinates so solve for the y intercept.

5= 4(2) + b

5 = 8 + b

-8   -8

b = -3

The equation will be   y = 4x -3  

We now have the equations y=4x -3   and  y = 3x -1  

To find that their solution is (2,5)  substitute one of the y-values into the other and solve for x then solve for y.  

4x -3 = 3x -1

    +3        +3

4x = 3x + +2

-3x   -3x

1x = 2

x = 2  

We know now that the value of x is true because the value of x is 2.Input it into one of the equations and solve for y.

y = 4(2) - 3

y = 8-3

y = 5

The solution is indeed (2,5) .

complete the statement.
(7m-2m2n)2
m = -6 and n = 2

Answers

The value of the given expression (7m - 2m²n)² = 34,596 at m = -6 and n = 2.

What is an expression?

An expression consists of variables, constants, and operators. Based on the number of terms in an expression, they are named linear, binomial, and so on.

Calculation:

The given expression is (7m - 2m²n)²

It is given that m = -6 and n = 2

To evaluate the given expression, the given values are substituted in it.

On substituting these values,

(7m - 2m²n)² = (7(-6) - 2(-6)²(2))²

⇒ (-42 - 4(36))²

⇒ (-42 - 144)²

⇒ (-186)²

∴ (7m - 2m²n)² = 34,596

Therefore, the required value of the given expression is 34,596.

Learn more about an expression here:

https://brainly.com/question/2215215

#SPJ1

if f(x)=3x^2+1 and g(x)=1-x, what is the vaue of (f-g)(2)?
[•12 •14 •36 •38]

Answers

Step-by-step explanation:

(f-g)(x) = f(x) - g(x).

When x = 2, f(2) - g(2) = 3(2)^2 + 1 - (1 - 2) = 12 + 1 + 1 = 14.

q(x) = 2x+5
f(x)= x^2+1
find (gof)(7)

Answers

Answer:105

Step-by-step explanation:

first find f(7)

(7)^2 + 1

=50

then put the answer for f(7) in g of x

2(50) + 5

=105

You are skiing down a mountain with a vertical height of 800 feet. The distance from the top of the mountain to the base is 1600 feet. What is the angle of elevation from the base to the top of the mountain?

Answers

Answer:

θ  = [tex]30^{0}[/tex]

Step-by-step explanation:

Since the vertical height of the mountain is 800 feet, and the distance from the top to its base is 1600 feet. Let the angle of elevation be represented by θ, applying the required trigonometric function;

Sin θ = [tex]\frac{opposite}{hypotenuse}[/tex]

Sin θ = [tex]\frac{800}{1600}[/tex]

        = 0.5

θ = [tex]Sin^{-1}[/tex] 0.5

θ  = [tex]30^{0}[/tex]

Thus, the angle of elevation from the base to the top of the mountain is [tex]30^{0}[/tex].

Jon borrows $4.50 from his friends to pay for snacks, and $13.65 from his sister to go to the movies. How much money does he reed to repay?

Answers

Jon needs to repay $18.15

Answer:

$18.15

Step-by-step explanation:

which of the following expression, when evaluated, equals an irrational number ?

Answers

Answer:

the answer for number 38 is H

Write 81% as a decimal.

Answers

Answer: 0.81

Step-by-step explanation: Think about all the way to the 100.

While hiking Mr.Condo burns 402 calories in 6 hours. how many calories does he burn in 60 minutes

Answers

In 360min(6hrs),402 calories burn.
In 1min,402/360 calories burn.
In 60min 402/360*60=67 calories burn.
Other Questions
Let sin(45)=0.7071.Enter the angle measure (), in degrees for cos()=0.7071.any value equivalent to 360n + , wheren is an integer A small business owner has decided to form a ______________ after deciding that she wants to maximize revenue, minimize taxes, give back to the community, and exercise total control over her business. Which number line have a point that is equal to point p? Check all that apply Sprinkler powder on the carron board _________ friction Write the improper fraction 33/10 as a mixed number the difference of a number x and 5 PLEASE HELP ASAP ITS DUE RIGHT NOW How do social factors bring about change in a country? will give brainliest to whoever answers first and/or correctly If a/\2+1/a/\2=11, then a+1/a=? Royalties must only be paid for the use of materials by private organizations.TrueFalse solve this please[tex]5x - 2(4x + 11) = 14[/tex] Listen to the audio and then answer the following questions feel free to listen to the audio as many times as necessary before answering the question why is Manuel worried When you have carefully checked all the facts and your attitudes and still find that theres just something about your supervisor thats causing a problem in your relationship, you should suspect that Evaluate the following function over the domain {0,1,2,3}. What is the range? g (x) = -2x^{2} + 2 Please hurry Which of these structures are found in both plant and animal cells?cell wall, mitochondria, ribosomesmitochondria, ribosomes, vacuolesvacuoles, chloroplasts, cytoplasmcytoplasm, cell wall, vacuoles Is 8/5 an irrational number? Which of the following molecules would form a micelle?a. NaClb.CH3COOHc. CH3COO^-NH^4+d. CH3(CH2)^10COO^-Na^+e. CH3(CH2)^10CH3 write the number 3/100 as a decimal. If it is an addition problem with different signs what do you do Frank has a sample of steel that weighs 80 grams. If the density of his sample of steel is 8 g/cm3, what is the samples volume? describe the application of rules